[Date Prev][Date Next][Thread Prev][Thread Next][Date Index][Thread Index]

Re: [obm-l] En: Putnam Question



Pelo visto, hoje alguém estava (des)inspirado para comentar as mensagens...

Perdoe-me, em que trecho do enunciado diz-se que há valor numérico para a
resposta? Vou ajudá-lo, eis o enunciado:

"Triangle ABC has an area 1. Points E, F, and G,lie respectively on sides BC
, CA, and AB, such that AE bisects BF at point R, BF bisects CG at point S,
and CG bisects AE at point T, Find the area of triangle RST."

Concordo que a área de ABC é dada, mas a questão não *pede* um valor
numérico para o triângulo RST, isso não está escrito em qualquer lugar. A
menos que, além de dizer que o exercício está resolvido errado, você possa
resolver corretamente, com ou sem valores numéricos....



----- Original Message -----
From: Johann Peter Gustav Lejeune Dirichlet
To: obm-l@mat.puc-rio.br
Sent: Sunday, March 14, 2004 3:12 PM
Subject: Re: [obm-l] En: Putnam Question


Nao pois a questao pede valores numericos, independente de lados ou
segmentos inerentes ao desenho...

=========================================================================
Instruções para entrar na lista, sair da lista e usar a lista em
http://www.mat.puc-rio.br/~nicolau/olimp/obm-l.html
=========================================================================